Q12

 
hamham
Thanks Received: 0
Forum Guests
 
Posts: 10
Joined: June 16th, 2013
 
 
 

Q12

by hamham Mon Nov 30, 2015 12:25 am

I chose D after POE, but I can't find support for this. Can anyone help me? :)
 
emossor
Thanks Received: 0
Vinny Gambini
Vinny Gambini
 
Posts: 4
Joined: October 23rd, 2015
 
 
 

Re: Q12

by emossor Wed Dec 02, 2015 9:16 pm

I did the same. Looking back at it afterwards I saw line 39-40 "some researchers took measures to prevent competitors from gaining access to materials they had created". Seems to be the closest thing to asserting legal control.

Also line 43 "mistakenly assume granting a patent implies granting the right to deny access". Now just because they wont deny access doesn't mean they wont allow open doors for everything. One can assume there is a possible middle ground such as conditional access based on the nature of research (look at non-commercial research... it is effectively a control over the use for a specific type of research).

That is my best shot at making sense of D.
 
Olivia James
Thanks Received: 0
Vinny Gambini
Vinny Gambini
 
Posts: 6
Joined: April 22nd, 2015
 
 
 

Re: Q12

by Olivia James Thu Dec 03, 2015 5:30 pm

Hello -

The proof for this question would most probably be found in paragraph 2 . Since the threatening aspects of patenting is expounded upon in paragraph 2.

I believe that support for D rests in the following statements -

1. L. 17-19 "These restrictions are seen as arising either from enforcement of a patent right or through operation of a contractual agreement."
2. This expounds upon the previous statement. - L. 23-27 "a patent holder may refuse to make such materials available to scientists conducting basic research unless a costly materials-transfer agreement or license is undertaken."


Hope this was helpful (and correct) -

Good Luck!
User avatar
 
ohthatpatrick
Thanks Received: 3806
Atticus Finch
Atticus Finch
 
Posts: 4661
Joined: April 01st, 2011
 
This post thanked 1 time.
 
 

Re: Q12

by ohthatpatrick Mon Dec 07, 2015 2:17 am

Nailed it again, Olivia. And once again, I shall play clean-up duty with the wrong answers.

On an open-ended (no keyword) question stem that uses words like "can be inferred / implies / suggests / most likely to agree", you're basically looking out for dangerous answer choice wording
strong, comparative, out of scope
and giving initial consideration to weak, aloof, obnoxious wording
can, may, might, not always, need not, not only, not all

(A) COMPARISON / OUT OF SCOPE -- No support for ranking policy makers vs. researchers on who favors new restrictions more.

(B) STRONG - "Most"? This idea is just also a fake opposite. Academics are worried that patent holders' concern over market position threatens the pursuit of basic research.

(C) STRONG - "Generally unable"? We know that more than 50% of these rebels can't obtain funding?

(E) STRONG - "a dearth"? There's a scarcity of highly educated biologists willing to teach?

Meanwhile, the correct answer (D) invites us in with its come hither weak language, "X is not the only way to do Y". All you need to support this answer is some other example of a way that a patent holder can assert legal control over its patented materials.

They could charge you money for using their precious cell lines in your research.

Hope this helps.